De digitale vraagbaak voor het wiskundeonderwijs

home |  vandaag |  gisteren |  bijzonder |  gastenboek |  wie is wie? |  verhalen |  contact

HOME

samengevat
vragen bekijken
een vraag stellen
hulpjes
zoeken
FAQ
links
twitter
boeken
help

inloggen

colofon

  \require{AMSmath} Printen

F(z)=Log z

Gegeven is f(z)= Log z.

Teken in het complexe vlak het beeld van
{z / 1/z/4} onder f.


Het volgende had ik gedaan:
Log z=ln /z/ + i Argz
Voor verschillende waarden waarvan geldt /z/=1 of /z/=4 heb ik het beeld bepaald en aan de hand daarvan het gevraagde beeld getekend. Maar dit was niet goed, maar hoe moet ik het dan aanpakken??

BVD
Tjen

Tjen
Student hbo - vrijdag 30 december 2005

Antwoord

De gegeven verzameling is de verzameling reiq met 1r4 en -pqp.
ln(z) is gedefineerd als ln(r)+iq, dus ln(z) is de verzameling a+bi met 0aln(4) en -pbp. Dit gebied is een rechthoek met hoekpunten (0,-p), (0,p) (ln(4),-p) en (ln(4),p).

Maar:
Ln(reiq)=ln(reiq)+2kip. (let op de hoofdletter)
Dat wil zeggen ln(z) is de hoofdtak van Ln(z).
Het beeld van de gegeven verzameling onder Ln(z) is dus een oneindige strook tussen a=0 en a=ln(4). De rechthoekjes zetten zich dus periodiek naar boven en naar onder voort.

Wie is wie?
Vragen naar aanleiding van dit antwoord? Klik rechts..!
zaterdag 31 december 2005



home |  vandaag |  bijzonder |  gastenboek |  statistieken |  wie is wie? |  verhalen |  colofon

©2001-2024 WisFaq - versie 3